PT39.S3.Q19-- Help!-- RC inference question

DarklordDarklord Alum Member
edited June 2020 in Reading Comprehension 586 karma

Hi,

So this question is really hard in my opinion-- I have been looking at this problem for quite a while and I still can't see why A is correct. Can anyone explain why A was correct here?

Thanks!
Best regards

Admin Note: https://7sage.com/lsat_explanations/lsat-39-section-3-passage-3-questions/

Sign In or Register to comment.